Foros-FIUBA Foros HostingPortal
 FAQ  •  Buscar  •  Wiki  •  Apuntes  •  Planet  •  Mapa  •  Eyeon  •  Chat
Preferencias  •  Grupos de Usuarios
Registrarse  •  Perfil  •  Entrá para ver tus mensajes privados  •  Login
Ver tema siguiente
Ver tema anterior

Responder al tema Ver tema anteriorEnviar por mail a un amigo.Mostrar una Lista de los Usuarios que vieron este TemaGuardar este Tema como un archivoPrintable versionEntrá para ver tus mensajes privadosVer tema siguiente
Autor Mensaje
nachito44
Nivel 6


Edad: 34
Registrado: 11 Jul 2008
Mensajes: 268

Carrera: Civil
CARRERA.civil.3.jpg
MensajePublicado: Dom Jun 21, 2009 12:49 pm  Asunto:  Problema de transformaciones lineales Responder citandoFin de la PáginaVolver arriba

Les dejo un ejercicio que me dejo dudas. Es el 3 d) de la practica 5.

Decidir si existe una transformacion lineal f que satisface las condiciones dadas; en caso afirmativo, si es unica, encontrar la expresion f(x).

f: R^3 --> R^3, f(0,1,1)=(1,2,3), f(-1,2,1)=(-1,0,1), f(-1,3,2)=(0,2,3)

Y aqui les dejo mis dudas. Primero de todo, analizo si son LI (0,1,1), (-1,2,1) y (-1,3,2) y veo que no. Entonces no puedo aplicar el teorema de que existiria una unica TL. Por ende paso a tomar la CL de un vector respecto de los otros dos y a aplicar f m.a.m en dicha expresion. Pero tomando por ejemplo (0,1,1), veo que no se verifica la CL, entonces aqui surgen mis dudas, que habria que hacer a continuacion:

¿Probar los otros casos o de una decir que no existe dicha combinacion lineal? Porque me dijeron que en caso de que si verificara, no tendria que hacerlo con los tres casos, sino que con uno alcanzaria, ¿Porque?. Porque se me ocurre a mi que si se tomaran dos vectores multiplos entre si y un tercero, en el caso de que se tomara el tercero y uno de los otros, podrian ser la CL del primero, pero no en el caso de que el tercero sea la CL de los otros dos.

A ver si pueden aclararme un poco el panorama.
Muchas gracias


Sagitario Género:Masculino Serpiente OfflineGalería Personal de nachito44Ver perfil de usuarioEnviar mensaje privado
Freddy
Nivel 8


Edad: 34
Registrado: 29 Oct 2008
Mensajes: 630
Ubicación: Lanús
Carrera: Sistemas
blank.gif
MensajePublicado: Dom Jun 21, 2009 4:28 pm  Asunto:  (Sin Asunto) Responder citandoFin de la PáginaVolver arriba

Me parece que te la estás complicando al gas..

Vos tenés:

f(0,1,1)=(1,2,3)
f(-1,2,1)=(-1,0,1)
f(-1,3,2)=(0,2,3)

Los tres vectores son L.D. (0,1,1) + (-1,2,1) = (-1,3,2).
Entonces, por definición de T.L, se tiene que cumplir que:

f( (0,1,1) + (-1,2,1) ) = f(0,1,1) + f(-1,2,1) = (1,2,3) + (-1,0,1) =

(0,2,4).

Sin embargo, f( (0,1,1) + (-1,2,1) ) = f(-1,3,2) = (0,2,3) [Esto lo tenés arriba], entonces no existe una t.l. definida de esa manera.


Capricornio Género:Masculino Serpiente OfflineGalería Personal de FreddyVer perfil de usuarioEnviar mensaje privadoEnviar emailMSN Messenger
nachito44
Nivel 6


Edad: 34
Registrado: 11 Jul 2008
Mensajes: 268

Carrera: Civil
CARRERA.civil.3.jpg
MensajePublicado: Dom Jun 21, 2009 7:49 pm  Asunto:  (Sin Asunto) Responder citandoFin de la PáginaVolver arriba

Muchas gracias por la respuesta. Igual justamente mis dudas vienen como dije a raiz del ejercicio pero son generales del tema, por eso es que las planteaba y tomaba el ejercicio como ejemplo. Porque no se si en el caso de que me diera LD, y no formaran una base por ende, planteando un caso de CL de un vector respecto a los otros dos y aplicando la funcion, y viendo que se cumple, bastaria para afirmar que existe dicha TL o habria que analizar todos los casos, porque justamente podria ser que unos dos vectores como dije resulten en otro, pero cambiando y tomando otros, no se obtenga, por lo que no se podria generalizar. Porque dependeria del caso que primero se me "ocurriese" trabajar, o sea en este se podria plantear 3 CL y justo aqui no ocurre, pero si podria ser, que dos generen uno pero otros no, por eso mis dudas conceptuales respecto del tema.
Gracias nuevamente


Sagitario Género:Masculino Serpiente OfflineGalería Personal de nachito44Ver perfil de usuarioEnviar mensaje privado
Bimba
Nivel 8


Edad: 35
Registrado: 13 Sep 2007
Mensajes: 587

Carrera: Química
CARRERA.quimica.3.jpg
MensajePublicado: Lun Jun 22, 2009 12:19 am  Asunto:  (Sin Asunto) Responder citandoFin de la PáginaVolver arriba

Creo que el tema de "cuánto tenés que probar", viene por el lado de cuántos te dieron LD.

Por ejemplo en este caso sólo tenías uno que era CL de los otros dos, entonces verificás ese (como explicó Freddy) y listo, no tenés más nada que probar. Si para tres vectores tuvieras que dos son CL de los otros entonces sí tendrías que probar con las dos CL y ver cómo dan los transformados.
O sea, no sé bien qué es lo que te dijeron de que si uno da entonces ya es TL, yo creo (y espero no equivocarme) que por más que uno te de igual la CL de los transformados, si para el otro (que te había dado LD) no verifica, entonces No es TL.
La idea es que tendrías un elemento del espacio de partida yendo a dos del de llegada, o en otras palabras, con dos imágenes. Si eso pasa No es función, y para el caso, tampoco transformación lineal.

Saludos


Leo Género:Femenino Dragón OcultoGalería Personal de BimbaVer perfil de usuarioEnviar mensaje privado
gonzaloi
Nivel 7


Edad: 34
Registrado: 06 May 2008
Mensajes: 398

Carrera: No especificada
argentina.gif
MensajePublicado: Lun Jun 22, 2009 12:19 pm  Asunto:  (Sin Asunto) Responder citandoFin de la PáginaVolver arriba

nachito44 escribió:
Muchas gracias por la respuesta. Igual justamente mis dudas vienen como dije a raiz del ejercicio pero son generales del tema, por eso es que las planteaba y tomaba el ejercicio como ejemplo. Porque no se si en el caso de que me diera LD, y no formaran una base por ende, planteando un caso de CL de un vector respecto a los otros dos y aplicando la funcion, y viendo que se cumple, bastaria para afirmar que existe dicha TL o habria que analizar todos los casos, porque justamente podria ser que unos dos vectores como dije resulten en otro, pero cambiando y tomando otros, no se obtenga, por lo que no se podria generalizar. Porque dependeria del caso que primero se me "ocurriese" trabajar, o sea en este se podria plantear 3 CL y justo aqui no ocurre, pero si podria ser, que dos generen uno pero otros no, por eso mis dudas conceptuales respecto del tema.
Gracias nuevamente


Esa afirmacion es falsa ....si se te cumple con una combinacion se te cumple con todas;si no se te cumple con una, no se te cumple con ninguna.
Fijate probando con los pocos ejercicios de este tipo que hay en la guia.


Sagitario Género:Masculino Serpiente OfflineGalería Personal de gonzaloiVer perfil de usuarioEnviar mensaje privado
nachito44
Nivel 6


Edad: 34
Registrado: 11 Jul 2008
Mensajes: 268

Carrera: Civil
CARRERA.civil.3.jpg
MensajePublicado: Lun Jun 22, 2009 9:51 pm  Asunto:  (Sin Asunto) Responder citandoFin de la PáginaVolver arriba

Muchas gracias a todos por las respuestas. Gonzaloi una pregunta mas:

Cuando se cumple, ya esta con que cumpla la combinacion lineal y la aplicacion posterior de la funcion es decir de transformacion lineal, pero en el caso de que no cumpla, no importa que no cumpla desde la transformacion o de la combinacion??

Por ejemplo, considerando (0,2,2), (0,4,4) y (1,2,3)... (0,2,2) y (1,2,3) son la combinacion lineal de (0,4,4), y lo mismo tomando (0,4,4) y (1,2,3) se obtiene (0,2,2) pero si se toma (0,2,2) y (0,4,4), es imposible que sean la combinacion lineal de (1,2,3)... en este caso por ejemplo: ¿Seria imposible que en las dos primeras partes, de que la combinacion lineal es posible, de que se verifique tambien la funcion?, porque en ese caso seria transformacion lineal, pero en el ultimo no pasaria lo mismo porque no se cumpliria ni la combinacion lineal, ¿Habria que analizar los tres? o ¿Es siempre lo mismo, que con un caso basta, mas alla de que si se cumple o no la combinacion? porque aunque en uno se cumpliera, ¿nunca se cumpliria la funcion de transformacion?

Perdon por tantas preguntas y gracias por aclararme el panorama


Sagitario Género:Masculino Serpiente OfflineGalería Personal de nachito44Ver perfil de usuarioEnviar mensaje privado
gonzaloi
Nivel 7


Edad: 34
Registrado: 06 May 2008
Mensajes: 398

Carrera: No especificada
argentina.gif
MensajePublicado: Mar Jun 23, 2009 3:50 pm  Asunto:  (Sin Asunto) Responder citandoFin de la PáginaVolver arriba

Me mariastes, jejeje. Es facil mira....para ver si una funcion existe, partiendo de condiciones que te impone el problema , tenes que fijarte solo dos cosas:(Tomo el ejercicio que esta al principio del thread)

1)Siempre para definir una funcion tengo que partir de una base que me genere todo el espacio de salida (en este caso R^3).
--->{(1,2,3) (0,2,3) (0,1,1)} no es una base,pues el primer o segundo vector es combinacion de los otros dos.Entonces hasta el momento podemos afirmar que existen infinitas funciones, ya que como dije tenemos que partir de una base, entonces tendriamos que agregar el vector que se nos ocurra para completar la base y asi poder definir la funcion ( en este ejercicio no voy a agregar un vector porque no me piden definir ninguna funcion)

2)fijarme que la combinacion lineal del vector que es ld (en este caso puedo tomar a (-1,2,1) o a (-1,3,2) ,tomemos al primero por ejemplo) cumpla con la transformacion lineal:

como dijimos ,tomamos a (-1,2,1) y dijimos que es combinacion lineal del resto, es decir...

(-1,2,1)=a(0,1,1)+b(-1,3,2) tal que (resolvemos) a=-1,b=1.Aplicamos f...
--->f(-1,2,1)=a.f(0,1,1)+b.f(-1,3-2) aplico la transformacion...
--->(-1,0,1) =a (1,2,3) + b (0,2,3) reemplazo a y b...
--->(-1,0,1)=(-1)(1,2,3) + (1) (0,2,3) distribuyo y resuelvo...
--->(-1,0,1)=(-1,0,0) (NO SON IGUALES)

Coclusion no existe NINGUNA T.L que cumpla con las condiciones establecidas.
Nota: en caso que se hubiece cumplido la ultima igualdad, existirian infinitas funciones,ya que como dije,para armar una funcion tenemos que partir de una base que me genere todo el subespacio de salida.
Nota2: podria haber tomado el (-1,3,-2) con ld del resto y hubice concluido lo mismo ya que no depende del vector que tomes sino de las variables (en este caso a y b)

Espero que me entiendas...sino, pregunta, que se viene el parcial.Salu2


Sagitario Género:Masculino Serpiente OfflineGalería Personal de gonzaloiVer perfil de usuarioEnviar mensaje privado
gonzaloi
Nivel 7


Edad: 34
Registrado: 06 May 2008
Mensajes: 398

Carrera: No especificada
argentina.gif
MensajePublicado: Mar Jun 23, 2009 4:04 pm  Asunto:  (Sin Asunto) Responder citandoFin de la PáginaVolver arriba

nachito44 escribió:
Muchas gracias a todos por las respuestas. Gonzaloi una pregunta mas:

Cuando se cumple, ya esta con que cumpla la combinacion lineal y la aplicacion posterior de la funcion es decir de transformacion lineal, pero en el caso de que no cumpla, no importa que no cumpla desde la transformacion o de la combinacion??


si no cumple desde la combinacion (antes de aplicar la transformacion digamos)es porque no es ld el vector que tomastes con el resto.Entonces no tenes que estudiar ese vector.

nachito44 escribió:


Por ejemplo, considerando (0,2,2), (0,4,4) y (1,2,3)... (0,2,2) y (1,2,3) son la combinacion lineal de (0,4,4), y lo mismo tomando (0,4,4) y (1,2,3) se obtiene (0,2,2) pero si se toma (0,2,2) y (0,4,4), es imposible que sean la combinacion lineal de (1,2,3)... en este caso por ejemplo: ¿Seria imposible que en las dos primeras partes, de que la combinacion lineal es posible, de que se verifique tambien la funcion?, porque en ese caso seria transformacion lineal, pero en el ultimo no pasaria lo mismo porque no se cumpliria ni la combinacion lineal, ¿Habria que analizar los tres? o ¿Es siempre lo mismo, que con un caso basta, mas alla de que si se cumple o no la combinacion? porque aunque en uno se cumpliera, ¿nunca se cumpliria la funcion de transformacion?

Perdon por tantas preguntas y gracias por aclararme el panorama


Solo tenes que analizar el vector que sea ld con el resto, tal como te indique arriba.Te estas haciendo mucho lio.


Sagitario Género:Masculino Serpiente OfflineGalería Personal de gonzaloiVer perfil de usuarioEnviar mensaje privado
nachito44
Nivel 6


Edad: 34
Registrado: 11 Jul 2008
Mensajes: 268

Carrera: Civil
CARRERA.civil.3.jpg
MensajePublicado: Mar Jun 23, 2009 4:32 pm  Asunto:  (Sin Asunto) Responder citandoFin de la PáginaVolver arriba

Muchas gracias. Entendi todo el lio que me habia hecho. Lo unico cuando dices de definir una funcion...

Cita:
Siempre para definir una funcion tengo que partir de una base que me genere todo el espacio de salida (en este caso R^3).
--->{(1,2,3) (0,2,3) (0,1,1)} no es una base,pues el primer o segundo vector es combinacion de los otros dos.


¿Te equivocaste y habia que considerar (0,1,1), (-1,2,1) y (-1,3,2)? ¿O esta bien que es (1,2,3) (0,2,3) (0,1,1)? Porque yo siempre analizaba el primer caso, los otros simplemente los utilizaba para ver si comprobaban.
Es la unica duda ahora, gracias a todos.


Sagitario Género:Masculino Serpiente OfflineGalería Personal de nachito44Ver perfil de usuarioEnviar mensaje privado
gonzaloi
Nivel 7


Edad: 34
Registrado: 06 May 2008
Mensajes: 398

Carrera: No especificada
argentina.gif
MensajePublicado: Mar Jun 23, 2009 9:43 pm  Asunto:  (Sin Asunto) Responder citandoFin de la PáginaVolver arriba

Me confundi de vectores , cuac


Sagitario Género:Masculino Serpiente OfflineGalería Personal de gonzaloiVer perfil de usuarioEnviar mensaje privado
nachito44
Nivel 6


Edad: 34
Registrado: 11 Jul 2008
Mensajes: 268

Carrera: Civil
CARRERA.civil.3.jpg
MensajePublicado: Mar Jun 23, 2009 9:56 pm  Asunto:  (Sin Asunto) Responder citandoFin de la PáginaVolver arriba

Menos mal, porque si no tenia nuevas dudas jaja. Muchas gracias


Sagitario Género:Masculino Serpiente OfflineGalería Personal de nachito44Ver perfil de usuarioEnviar mensaje privado
Mostrar mensajes de anteriores:      
Responder al tema Ver tema anteriorEnviar por mail a un amigo.Mostrar una Lista de los Usuarios que vieron este TemaGuardar este Tema como un archivoPrintable versionEntrá para ver tus mensajes privadosVer tema siguiente

Ver tema siguiente
Ver tema anterior
Podés publicar nuevos temas en este foro
No podés responder a temas en este foro
No podés editar tus mensajes en este foro
No podés borrar tus mensajes en este foro
No podés votar en encuestas en este foro
No Podéspostear archivos en este foro
No Podés bajar archivos de este foro


Todas las horas son ART, ARST (GMT - 3, GMT - 2 Horas)
Protected by CBACK CrackerTracker
365 Attacks blocked.

Powered by phpBB2 Plus, phpBB Styles and Kostenloses Forum based on phpBB © 2001/6 phpBB Group :: FI Theme :: Mods y Créditos

Foros-FIUBA está hosteado en Neolo.com Cloud Hosting

[ Tiempo: 0.5028s ][ Pedidos: 20 (0.4216s) ]